LSAT and Law School Admissions Forum

Get expert LSAT preparation and law school admissions advice from PowerScore Test Preparation.

User avatar
 Dave Killoran
PowerScore Staff
  • PowerScore Staff
  • Posts: 5852
  • Joined: Mar 25, 2011
|
#46252
Complete Question Explanation
(The complete setup for this game can be found here: lsat/viewtopic.php?t=14384)

The correct answer choice is (E)

Answer choice (A) is incorrect because K and N cannot be assigned to the same team per the fourth rule.

Answer choice (B) is incorrect because K must be assigned to a middle position in accordance with the second rule.

Answer choice (C) is incorrect because according to the third rule, if L is assigned, L must be assigned to team 1.

Answer choice (D) is incorrect because from the fifth rule P and Q cannot be assigned to the same team.

Answer choice (E) is the correct answer choice.

Get the most out of your LSAT Prep Plus subscription.

Analyze and track your performance with our Testing and Analytics Package.